Download 4/ Adjunctive therapy for sepsis

Survey
yes no Was this document useful for you?
   Thank you for your participation!

* Your assessment is very important for improving the workof artificial intelligence, which forms the content of this project

Document related concepts

Prenatal testing wikipedia , lookup

Infection wikipedia , lookup

Dental emergency wikipedia , lookup

Patient safety wikipedia , lookup

Canine parvovirus wikipedia , lookup

Focal infection theory wikipedia , lookup

Infection control wikipedia , lookup

Intravenous therapy wikipedia , lookup

List of medical mnemonics wikipedia , lookup

Transcript
INTRODUCTION
Intensive care units were established to provide support of vital organ function during a time
of life-threatening physiologic dysfunction. Their success is reflected in the fact that the vast
majority of patients admitted to an ICU survive an illness that half a century ago would have
been rapidly lethal. However, in the successes of intensive care lie the roots of its greatest
unsolved challenges.
Sepsis is a lethal disease process, with mortality rates of 25% to 60% or higher, depending
on the severity of the illness, but also varying substantially from one country to the next. By
convention, the word sepsis is used to describe the syndrome of systemic inflammation
when infection is the cause, while the systemic inflammatory response syndrome or SIRS
denotes that process without reference to its causative trigger. However, it may be difficult
to determine whether infection is truly present, and if it is, whether it is the cause, or the
consequence of the deterioration that is evolving. The word is used in this module in a
looser clinical sense, to denote a syndrome that is commonly, but not always caused by
infection, but that heralds significant risk to the patient, and tremendous challenges to the
clinician. Nevertheless, when sepsis is suspected it is crucial to look for infection, as the
rapid and effective treatment of infection improves outcome.
Multiple organ dysfunction syndrome or MODS is responsible for 80% of all ICU deaths,
and evolves in the wake of a syndrome of systemic inflammation. Sometimes inflammation
is a consequence of a sterile insult such as pancreatitis, but more often, it is the result of
infection and sepsis, acquired in the community or complicating the course of a complex
illness.
A general overview of the problems of sepsis and MODS, and the evolution of ideas in their
description and management, can be found in the following references.
Bone R, Balk R, Cerra F, Dellinger RP, Fein A, Knaus W et al. : Definitions for sepsis and organ failure and guidelines for the
use of innovative therapies in sepsis. Chest 1992; 101: 1644-1655.PMID 1303622
What is sepsis?
Death for patients admitted to an intensive care unit is rarely a direct effect of the disorder
that led to ICU admission. Rather, critically ill patients die as a consequence of a syndrome
of secondary organ system disorders known as the multiple organ dysfunction syndrome
(MODS). For the patient admitted with pneumonia, the infection is treated, but after a
course characterised by renal failure, persistent hypotension, failure to wean from the
ventilator, and recurrent bouts of nosocomial infection often with relatively low virulence
organisms, the patient may succumb weeks later. A similar course follows admission for
very divergent illnesses — acute pancreatitis, ruptured abdominal aortic aneurysm,
refractory congestive heart failure, or decompensated liver cirrhosis.
The terminology conventionally used to describe the septic patient is confusing, and
attempts to simplify or clarify these concepts really have not accomplished their objectives.
This confusion arises from both the complexity of the disease, and our intellectual
sloppiness in trying to describe it.
The word 'sepsis' is attributed to Hippocrates (460 – 370 BC). In Hippocrates´ view, living
things died and were transformed through one of two basic, but opposite
processes. Pepsis denoted tissue breakdown that was restorative and life-giving, and was
exemplified by the digestion of food, or the fermentation of grapes to produce wine. Sepsis,
on the other hand, was tissue breakdown associated with disease, decay, a foul smell, and
death – for example, putrefaction, infection of wounds, and the vapours arising from
stagnant swamps. The advent of the microscope and the work of Louis Pasteur
demonstrated that the processes of sepsis arose through the actions of micro-organisms;
intriguingly, those of pepsis – whether fermentation or peptic ulceration – are also a product
of microbial activity.
Sepsis to the clinician has a somewhat different connotation. It describes an acute clinical
syndrome that is quite variable and often non-specific in its clinical presentation, but that
suggests the presence of uncontrolled infection, and the threat of imminent clinical
deterioration.
T HINK
How would you define sepsis? Must a patient have a culture-proven infection to be
called septic, or is the essence of the syndrome the clinical response of the host? Do
all patients with infection have sepsis? Do all patients with sepsis have infection?
Write down five physiological or laboratory abnormalities that you believe to be
most characteristic of sepsis. Now ask a colleague to do the same, and compare
your lists? Are they the same? Is there a single clinical syndrome that we might
call sepsis?
The definition of sepsis is frustratingly elusive, because sepsis is not so much a syndrome
or a disease, as it is a state of profoundly deranged host-microbial homeostasis. In health,
humans, like all multicellular creatures, live in a state of symbiosis with the microbial world.
We consume micro-organisms when we eat, they colonise our mucosal surfaces in health,
and as mitochondria, they even power the fundamental processes of cellular metabolism
that allow us to live. Bacteria in the gut aid in the digestion of food, stimulate intestinal
epithelial development, and inhibit the growth of exogenous and potentially more virulent
organisms. In turn, we provide nutrients and a favourable environment for growth, and a
state of mutually beneficial tolerance ensues. In sepsis, however, this state is disrupted:
micro-organisms become a threat to the host, and the host activates a series of processes
to kill the micro-organism. Normal patterns of microbial colonisation are altered, as are
normal patterns of the host antimicrobial response. Infection becomes both the cause of
sepsis and a manifestation of the disorder; the host response, heralded by inflammation,
becomes both evidence of the disorder, and a cause of the tissue destruction that results.
However, concepts such as this are perhaps too esoteric to guide clinical decision-making,
and instead, we have, over time, developed a series of generally agreed upon, though
imperfect, definitions for the clinical syndrome of sepsis.
Infection is a microbial phenomenon: the invasion of normally sterile host tissues by viable
micro-organisms. Infection both triggers and exacerbates the host response, and should
always be sought and treated. For example, patients with sterile pancreatic necrosis may
have MODS but infected necrosis greatly increases the risk of death. Similarly, burn
patients exhibit SIRS but most deaths are due to infection of the burn wound.
Microbial invasion typically evokes a response in the host that may be local, in which case
the local signs of inflammation – rubor, calor, dolor, tumor, and functio laesa – are present,
or it may besystemic. Sepsis, therefore, is defined as the systemic host response to
invasive infection.
However, it will be apparent that just as local signs of inflammation may arise from sterile
insults – a foreign body, an allergic reaction, or an autoimmune disease such as arthritis, for
example, so too systemic inflammation may result from a sterile insult – multiple trauma,
burns, blood transfusion, a drug reaction, pancreatitis, or thrombotic thrombocytopenic
purpura, to name a few. Thus the term thesystemic inflammatory response syndrome
(SIRS) was coined to denote clinical manifestations of systemic inflammation, independent
of cause; in this model, sepsis is present when SIRS arises as a consequence of infection
from various sources, such as lung, abdomen, urinary tract, (solid blue circles in figure,
below).
Overlap between infection,
SIRS and sepsis
A moment´s thought will disclose a further nuance that must be considered. A systemic
inflammatory response may be desirable and beneficial to the host when infection is
present. Yet, clinicians generally use the word sepsis to denote a state that is perceived as
a threat. Thus the severity of sepsis can be graded. Severe sepsis is sepsis in association
with hypoperfusion and mediator-induced organ dysfunction, an intuitively undesirable state,
while septic shock is severe sepsis of sufficient severity that perfusion is even more
profoundly jeopardised, vaso-regulatory mechanisms being overcome and tissues
becoming ischaemic. Some authors have proposed terms such as severe
SIRS anddistributive or vasodilatory shock to describe the equivalent gradations in the
patient without infection. Regardless of terminology, it is clear that each represents
increasingly more profound derangements in systemic homeostasis, reflected in an
increasing rate of mortality.
N OTE
The terminology of sepsis is inherently controversial and imprecise, but the
underlying concepts are not.The acute clinical syndrome – however defined or
characterised – identifies a patient at risk of clinical deterioration or death, and
must prompt the clinician to institute adequate resuscitative measures rapidly.
The syndrome reflects the response of the host, not the cause: it is critical that a
diagnosis be established, appropriate therapy be instituted, and unnecessary
interventions be minimised.
The immediate threat faced by the septic patient is not only the uncontrolled growth of
bacteria, but also the consequences of the systemic inflammatory response on oxygen
delivery to the tissues. In the most advanced stages, septic shock is said to be present – a
state in which oxygen availability is compromised, normal oxidative metabolism at the
cellular level is jeopardised, and anaerobic metabolism occurs. This state is characterised
biochemically by the release of lactate from the cells; although elevated serum lactate levels
are neither sensitive nor specific for shock.
Altered cellular metabolism results in altered cellular function, and even in cell death as a
consequence of necrosis or apoptosis. Apoptosis is a more physiologic and controlled
process of programmed cell death, however, those programmes may be heightened due to
inflammatory stimulation.
The biochemical processes of sepsis are enormously complex – the consequence of the
coordinated interaction of literally hundreds of host-derived mediator molecules including
cytokines, the complement and coagulation cascades, vasoactive mediators such as kinins,
prostaglandins, acute phase reactants, and short-lived intermediates of oxygen and
nitrogen. However, the resultant effects on blood flow and oxygen delivery to the tissue can
be readily understood as the consequence of four key acute changes, described below.
Vasodilation
Vasodilation involving small arterioles and nutrient vessels reflects the presence of
mediators and the dysfunction of compensatory mechanisms. A number of cytokines can
induce the expression of an enzyme known as inducible nitric oxide synthase (iNOS) in
vascular endothelial cells. iNOS catalyses the conversion of the amino acid arginine to
citrulline, in the process generating a molecule of nitric oxide. Nitric oxide is a potent smooth
muscle relaxing agent, and causes local vasodilatation – indeed this property accounts for
the pharmacologic effects of such classical vasodilatory agents as nitroglycerin and
nitroprusside. This may be a local process that directly opposes sympathetic stimulation.
Vasodilatation drastically increases the diameter of the vascular tree, reducing resistance,
inducing relative hypovolaemia (as the volume required to fill the tree is significantly
increased) and therefore lowers the effective blood pressure. Moreover, the loss of normal
microvasculature resistance results in accelerating the passage of blood through the
capillary beds, and therefore reduces the time available for passive unloading of oxygen
from the saturated red.
Loss of endothelial barrier function
Loss of integrity of the endothelial barrier – a consequence of disruption of the endothelial
tight junctions and loss of endothelial cells – results in loss of proteins and fluid into the
interstitium. This further decreases the effective intravascular volume. Moreover, the
resulting oedema aggravates cellular hypoxia by increasing the distance between the
erythrocyte in the capillary, and the adjacent cells, and so increasing the distance that
oxygen must diffuse to reach the cell.
Occlusion of capillaries
Occlusion of capillaries by thrombi, activated leukocytes, and aggregates of erythrocytes,
whose capacity for deforming during passage through the microvasculature has been
reduced, significantly impairs perfusion. Oxygenated blood bypasses these occluded
capillaries (shunt), failing to unload the oxygen and therefore increasing the local tissue
oxygen deficit.
Intravital sidestream dark
field images of the
sublingual
microcirculation:
capillaries with red cells
flowing (normal, upper image)
and obstructed with minimal
flow (sepsis, lower image).
Images courtesy of C Ince,
Amsterdam, the Netherlands
For further information, see the following references.
Impaired myocardial contractility
Impaired myocardial contractility as a consequence of poorly characterised myocardial
depressant factors further affects compensation. Reduced myocardial contractility is readily
demonstrable in animal models and in septic humans; its biologic basis is poorly understood
and probably multifactorial. Moreover its significance is uncertain, since the cardiac output
in sepsis is characteristically increased, and clinical evidence of impaired cardiac output
commonly reflects inadequate fluid resuscitation.
The net consequence of these abnormalities is the classical haemodynamic profile of
resuscitated sepsis: tachycardia, peripheral oedema, a hyperdynamic circulation (assuming
adequate fluid resuscitation), warm extremities, and hypoperfusion characterised by
elevated mixed venous oxygen saturation.
What is the pathologic basis for each of the following clinical features of severe sepsis or septic
shock?
 Tachycardia
 Tachypnoea
 Pyrexia
 Leukocytosis


T HINK
A reduced systemic vascular resistance
An increased mixed venous oxygen saturation
The classical signs of local inflammation – rubor, tumor, calor, dolor, and functio laesa
– have their systemic counterparts in patients with systemic inflammation. What are
they? Why is each of these beneficial to the host in the setting of local inflammation?
Why is each detrimental in systemic inflammation?
Recognising sepsis in the critically ill patient
Earlier consensus statements suggested that sepsis could be recognised by relatively nonspecific physiologic alterations thought to be characteristic for SIRS and including
tachycardia (a heart rate >90 beats per minute), tachypnoea (a respiratory rate >20 per
minute or the need for mechanical ventilation), hypo- or hyperthermia (a core temperature
<36.0 °C or >38.0 °C), and leukopenia or leukocytosis (a white blood cell count
<4000/mm3 or >11 000/mm3). While each of these reflects the physiologic abnormalities that
accompany the onset of systemic inflammation, they are neither specific for the process,
nor comprehensive in encompassing the acute derangements that signal sepsis. What is
more important from a clinical perspective is sensitivity to an acute change in clinical status
that denotes the systemic derangements of sepsis. In fact a large number of acute changes
in systemic physiologic homeostasis may be the early signs of the clinical syndrome of
sepsis (see reference below).
Sepsis is not a disease, but a syndrome whose evolution should prompt the clinician to look
for a treatable cause. The list of possible causes of SIRS is extensive (See Task 3
).
Infection is common, important, and generally readily treatable. For this reason it should
always be considered first. However, the clinician must also consider such causes as tissue
ischaemia, injury, non-infective inflammatory disorders such as pancreatitis or systemic
lupus erythematosus, drug or transfusion reactions, and other immunologically-mediated
processes such as thrombotic thrombocytopenic purpura. An educated team triage utilising
simple tools or reminders of SIRS and possible infections will facilitate the constant
vigilance required. If clinical monitoring systems are employed, simple alerts remind
clinicians of potential inflammatory processes.
N OTE
The diagnosis of infection in a patient admitted from the community is usually
straightforward, whereas establishing a definitive diagnosis of nosocomial
infection may be much more challenging. As a general rule, early and
appropriate treatment of community-acquired infection has a greater impact on
outcome. For more information see the PACT module on Severe infection
The early recognition of sepsis is not primarily the diagnosis of a specific disease, but the
recognition that a patient is ill and in danger of acute deterioration, and that immediate
intervention is needed to:



Restore haemodynamic stability and tissue perfusion
Determine the cause, and reverse or correct it
Institute appropriate physiologic support to prevent further tissue injury.
Let's now consider how these goals are accomplished.
2/ RESUSCITATION AND HAEMODYNAMIC SUPPORT OF THE SEPTIC PATIENT
The most immediate threat facing the septic patient is tissue hypoxia resulting from
inadequate oxygen delivery (DO2) in the development and course of septic shock, defined
by sepsis plus hypotension. At the cellular level, a lack of oxygen causes the cell to switch
from aerobic to anaerobic metabolism. This shift is accompanied by increased production of
lactate – the metabolic by-product of anaerobic metabolism – but also by altered cellular
metabolism, or even cell death, because of inadequate energy generation.
Multiple factors contribute to tissue hypoxia in sepsis:




Peripheral vasodilatation resulting in an increased intravascular diameter
requiring a higher vascular volume
Increased capillary permeability with leakage of protein-rich intravascular fluid
into the interstitium
Maldistribution of blood flow at the capillary level – secondary to microvascular
thrombosis, to capillary plugging by platelet thrombi or aggregates of red cells
and leukocytes, or to altered vasomotor tone – resulting in shunting of
oxygenated blood to the venous side of the circulation
Myocardial depression secondary to poorly characterised circulating factors.
How is oxygen carried in the blood transferred to cells adjacent to capillaries? List three factors
that might impede cellular uptake of oxygen in sepsis.
Recognising the perfusion defect of sepsis
Impaired tissue perfusion in sepsis in the development and course of septic shock is
recognised through its physiologic consequences; the manifestations are many, though
variable from one patient to the next.
Any acute change in normal haemodynamic homeostasis that is not readily
explained by another obvious diagnosis (for example, haemorrhage, myocardial
infarction, intoxications or poisoning) should prompt you to consider the
possibility of septic shock, particularly when there is suspicion for infection.
Tachycardia, a common finding in sepsis, is in part, a reflexive response to a relative
intravascular volume deficit. Physical examination of the patient with unresuscitated septic
shock reveals signs of peripheral hypoperfusion, a rapid, weak pulse, cool extremities, and
tissue pallor and cyanosis.
Reduced intravascular volume is also reflected in reduced pressure in the venous side of
the circulation. Under normal circumstances, the right atrial or central venous pressure
(CVP) is 0 to 5 mmHg, and the pulmonary capillary occlusion (or wedge) pressure (PAOP
or PCWP) is 5 to 12 mmHg when the plateau of the cardiac function curve relating cardiac
output to filling pressure is approached; lower levels are highly suggestive of an
intravascular volume deficit and of fluid responsiveness, i.e. a rise in cardiac output upon a
fluid challenge. For more information see the PACT module on Haemodynamic
monitoring
.
In the absence of factors that modify the distribution of blood in the microvasculature or the
normal relationship between the capillaries and cells in the immediately adjacent tissues,
reduced oxygen delivery results in increased oxygen extraction (VO2/DO2). This is a
compensatory mechanism that maintains oxygen consumption, as blood moves more
slowly through the microvasculature, and the gradient down which oxygen diffuses
becomes steeper. When oxygen extraction has reached a maximum, further lowering of
DO2 reduces VO2 below tissue needs. See the PACT module on Hypotension for more
information
.
When oxygen extraction increases, the oxygen saturation of the haemoglobin of venous
blood returning to the heart is lower than its normal 70% thus directly reflecting an
increased release of oxygen and providing us with a measurement that indicates poor
oxygen delivery. While the most representative sample of venous blood from the entire
body is the mixed venous blood in the pulmonary artery (as it evaluates the mixing of total
body including coronary sinus and cerebral blood), a sample drawn from the superior vena
cava or right atrium through a central venous catheter provides a reasonably reliable
estimate of the extent to which haemoglobin has become deoxygenated in the peripheral
tissues. Low central venous saturations indicative of inadequate resuscitation were well
demonstrated in early sepsis in the following study.
Following resuscitation, the central venous saturation rises for reasons discussed above.
Decreased tissue perfusion can be detected indirectly as metabolic acidosis, or more
directly, as an increased serum lactate level.
Reduced tissue perfusion results in acute alterations in organ function (see Task 5
).
This altered function is apparent in the kidney, where reduced blood flow results in reduced
urine production (oliguria) and increasing serum creatinine (see the PACT module on
Oliguria and anuria
). It is also seen in the cardiovascular system as persistent
tachycardia and hypotension (shock), and in the central nervous system, as confusion.
Hepatic hypoperfusion can result in hepatocellular injury, reflected in elevated levels of
hepatic transaminases. Pulmonary dysfunction is manifest as hypoxaemia due to
ventilation–perfusion mismatching, despite increased oxygen support.
The clinical and biochemical manifestations of tissue hypoperfusion are, as a rule, readily
apparent, and often multiple. A serialised measure of serum lactate in conjunction with
central or mixed venous monitoring facilitates evaluation of benefits of treatment. Their rapid
correction is critical.
Cardiovascular support in sepsis
The mainstay of cardiovascular support of the septic shock patient is the rapid
administration of intravenous fluids with the goal of restoring haemodynamic stability. The
choice of fluid is much less important than the amount. Large randomised trials, or
systematic syntheses of smaller studies, have failed to provide convincing evidence for the
superiority of crystalloids or colloids during fluid resuscitation.
Adequate intravenous access is established, and normal saline, Ringer´s lactate, or another
balanced electrolyte solution is rapidly administered, with the goal of giving a litre or more of
fluid within the first 30 minutes. Initial assessment of the response to resuscitation entails
monitoring of vital signs and urine output, and so a urinary catheter provides the clinician
with a tool to evaluate the dynamic response to intravenous fluids. Restoration of a normal
blood pressure and heart rate, and maintenance of a urinary output greater than 0.5
ml/kg/hr (adults) suggest that initial resuscitative measures have been successful. But what
if they aren't? More invasive strategies must be used to optimise tissue perfusion. The best
validated of these, and the approach adopted by the Surviving Sepsis Campaign, is the
algorithmic approach popularised by Rivers (below).
Treatment algorithm of
septic shock
First, continue to provide intravenous fluids, but insert a central venous catheter to monitor
the central venous pressure and the oxygen saturation of the central venous blood. Fluids
are given rapidly until the CVP is at least 6-8 mmHg, but continued improvement may occur
as the CVP is pushed to even higher levels, and so it is advisable to continue fluid
resuscitation as long as the blood pressure, urine output and, more specific measures of
fluid responsiveness respond, and pulmonary gas exchange is not significantly jeopardised.
T HINK
Optimising fluid resuscitation requires maximising the benefits of further fluids, while
minimising the risk –titrating therapy to the response of the individual patient.
List three physiologic responses that suggest your patient is responding adequately.
List three physiologic responses that suggest your patient is not responding
appropriately, but may in fact be experiencing harm.
For the next six patients with central venous catheters in place, and to whom
you are administering fluid, record the total amount of fluid you have given, the
CVP level at which you decided to stop administering fluids, and the time it took
to reach this goal from the time that resuscitation began. Are the responses
similar in all patients? How do your actions compare with the recommendations
of the Surviving Sepsis Campaign?
If the blood pressure remains low despite adequate volume resuscitation, then
vasopressors should be administered, and dosed so that the mean arterial pressure is
raised to about 65 mmHg. This target seeks to maximise vital organ perfusion, while
minimising the adverse consequences of the resulting vasoconstriction. Either
norepinephrine or dopamine are the vasopressor agents recommended by the Surviving
Sepsis Campaign but there is no compelling evidence that one is superior to the other.
Other vasoactive agents such as epinephrine (or perhaps vasopressin) might be equally
effective. Practically speaking, it is best to use the agent whose dosing and side-effect
profile you and your team are most comfortable with.
An increase in blood pressure does not necessarily result in increased tissue perfusion, and
perfusion, not pressure, is our therapeutic objective. The adequacy of global tissue
perfusion can be inferred by measuring the ScvO2, either with a specially designed catheter,
or simply by blood gas analysis of a sample of blood withdrawn through the CVP catheter.
The objective is to increase the ScvO2 to at least 70%, and this objective is achieved by one
or both of two strategies. First, an inotropic agent such as dobutamine is administered at a
dose of 5 µg/kg/min, and the dose titrated upwards based on the response of the central
venous oxygen saturation. High doses of dobutamine have been associated with increased
mortality, and so the total dose should probably not exceed 20 µg/kg/min. Second, if the
haemoglobin level is low (less than 4.96 mmol/l; 8.0 g/dl), packed red cells can be
transfused to raise the level to 6.2 mmol/l (10 g/dl).
Most patients will respond to these measures; those who do not pose
a formidable challenge for the intensivist and the critical care team,
and one for which approaches must be systematically clinical and
empirical. Moreover their risk of death is high. A reasoned approach
to this situation would consider diagnostic or iatrogenic error. First,
re-evaluate the clinical situation. Is the failure to respond a function of
refractory septic shock, or has another unrecognised complication
occurred; for example, tension pneumothorax following insertion of a
central catheter, severe pulmonary oedema resulting from aggressive
If the patient does not
respond as expected,
reconsider the possibility of
another diagnosis
resuscitation, inappropriate/inadequate antimicrobial therapy or acute
myocardial infarction with cardiogenic shock.
Once these are ruled out, consider the clinical context is aggressive therapy likely to result
in an acceptable quality of life for the patient, or are you simply engaging in a battle of wills
with what is clearly an irreversible process? Is the therapeutic objective reasonable? Set
your sights lower, and consider accepting a mean arterial pressure of 55, or even 50
mmHg, if trying to increase the pressure proves unsuccessful; consider a further fluid
challenge if the CVP is high but oxygenation acceptable. Before you initiate a further
intervention, define a therapeutic target, and decide what you must measure in order to
evaluate whether you are achieving your objectives.
A 67-year-old man is admitted with septic shock following an extensive small bowel resection for
intestinal infarction secondary to a superior mesenteric artery embolism. Following your initial
resuscitation, he is tachycardic with a heart rate of 136/min, and hypotensive with a blood pressure
of 76/40 mmHg; the central venous pressure is 12 mmHg, and the ScvO2 is 72%. He has produced
20 ml urine in the last two hours. His haemoglobin is 6.5 mmol/l (10.6 g/dl), and he is receiving
norepinephrine at a dose of 20 µg/min. He is mechanically ventilated, and his SaO2 is 96% with an
FiO2 of 0.6, and PEEP of 12 cmH2O.
Describe three potential interventions you might employ to manage this man, and the specific
parameters you will use to evaluate whether you have been successful.
3/ IDENTIFICATION AND CONTROL OF THE SOURCE OF INFECTION
Once resuscitation has been started, the next priority in the management of the septic
patient is to establish a diagnosis and to institute appropriate measures to treat the inciting
infection – specifically, performing appropriate cultures, thereafter starting empiric
antibiotics directed against the probable infecting pathogen(s) and undertaking source
control to eradicate a discrete infective focus.
Diagnosing infection in the acutely ill patient
Establishing an infective diagnosis in the acutely ill patient may be quite straightforward, but
is often enormously challenging. Our objective is to answer, as reliably and rapidly as
possible, three questions:



Is my patient infected?
What is the site from which the infection is arising?
What is/are the infecting pathogen(s)?
Is the patient infected?
The initial history and physical examination may establish a diagnosis of infection. In the
patient presenting to the emergency department with an acute illness, fever, malaise, and a
new cough productive of purulent sputum, this suggests the diagnosis of communityacquired pneumonia, while acute abdominal pain with findings of peritoneal irritation
suggests intra-abdominal infection. Physical examination may provide other evidence of a
localised infective process – a necrotising soft tissue infection, infected joint or head and
neck infection, for example. Recent hospital admission and indwelling catheters, among
other examples should alert the clinician to other possible infections.
Laboratory investigations can support the presumptive diagnosis of infection. An increased
white blood cell count is often present, however life-threatening infection can present with
leukopenia. Although an increased percentage of immature forms should be present in
either instance, the discriminatory ability of leukocyte changes to differentiate infective and
non-infective causes of fever is poor. Some studies have shown increased levels of Creactive protein to have a better discriminatory capacity, with an odds ratio of 5.4 for the
diagnosis of infection in patients with clinical signs of systemic inflammation. Increased
procalcitonin (PCT) levels provide even greater diagnostic accuracy, with an odds ratio of
15.7; procalcitonin has shown particular efficacy in discriminating infective from noninfective aetiologies in patients presenting to the emergency department with acute
respiratory symptoms. Other biomarkers of infection – for example soluble TREM-1 for the
diagnosis of pneumonia and endotoxin activity levels for the diagnosis of Gram-negative
infection – show diagnostic promise but experience is limited.
Culture evidence of tissue invasion currently represents the gold standard for the diagnosis
of infection. Specimens should be obtained prior to initiating antibiotics to maximise the
diagnostic yield. Positive blood cultures provide objective evidence of systemic
dissemination of a micro-organism; however the yield of blood culture positivity is variable,
depending on the nature of the infection as well as the methods of sampling. Following
adequate skin preparation, at least two samples should be taken from a peripheral site,
inoculating at least 10 ml blood into each specimen bottle. Positive cultures, taken using
sterile technique, of normally sterile tissue fluid – pleural or peritoneal fluid, for example –
also provide conclusive evidence of infection. Cultures obtained from a surface are less
reliable, for the differentiation of normal or pathologic colonisation from invasive infection is
difficult. Positive sputum or urine cultures in intubated or catheterised patients may reflect
colonisation of the tracheobronchial tree or lower urinary tract respectively; the use of
quantitative culture techniques, and invasive interventions such as bronchoscopy to obtain
specimens from a distal source, can improve the reliability of culture data.
Which of the following has not been shown to reduce the rate of contamination (false positivity)
during the performance of blood cultures?
a/ Skin preparation with chlorhexidine rather than povidone-iodine
b/ Aspiration of blood through a central catheter using full sterile technique
c/ Changing the needle prior to inoculating the specimen into a culture bottle
d/ Disinfecting the stopper on the culture bottle prior to inoculation
Definitive identification of the isolated microbial species, and evaluation of its sensitivity
profile to common antibiotics, typically involves a delay of at least two days. An earlier
presumptive microbial diagnosis can be made using a Gram stain (particularly of CSF or
urine samples), which can provide information on the class of organism within an hour.
When the suspected infection is a nosocomial infection arising in a hospitalised patient, the
diagnosis is particularly challenging. For this reason, it is probably clinically more accurate
to think of the probability that infection is present, rather than to try to definitively establish
or rule out infection. In other words, a decision to treat with antibiotics is made not on the
basis of a diagnosis of infection, but rather on the probability that infection is the cause of
the presenting clinical syndrome. Therefore, after cultures have been collected, a broadspectrum antibiotic should be considered, primarily based on the likely source of the
infection and knowledge of local pathogens.
For the next ten patients you are called on to evaluate for suspected sepsis,
write down your initial clinical impression of the probability of infection (very
unlikely, unlikely, neutral, likely, very likely) and the presumptive infective focus.
Repeat this evaluation three days later, using the results of laboratory,
radiographic, and microbiological investigations to guide your reassessment.
How accurate was your initial assessment? And don't be discouraged if it is
imperfect. Diagnostic inaccuracy is one of the intrinsic challenges of managing
the septic patient.
If the initial clinical assessment, augmented by the results of rapidly available investigations,
suggests that infection is likely to be present, then empiric antibiotic therapy should be
started expeditiously, guided by knowledge of local resistance patterns and the probable
infective focus. Delays in initiating therapy when severe sepsis or septic shock is present
have been associated with a time-dependent increase in the risk of death (figure below).
Equally, escalation of antibiotic therapy should be considered if the patient fails to respond
or if microbiological studies suggest this is necessary. Lastly, de-escalation must be
appropriately invoked when culture results indicate that treatment with a narrower spectrum
antibiotic is possible.
Delay between onset of
hypotension and the
initiation of antimicrobials
versus mortality
From Kumar A, Roberts D,
Wood KE, Light B, Parrillo JE,
Shrama S et al. Duration of
hypotension before initiation
of effective antimicrobial
therapy is the critical
determinant of survival in
human septic shock. Crit
Care Med 2006; 34(6): 15891596. PMID 16625125
What is the site of the infection?
Determining the site from which the infection is arising enables the clinician to refine the
prediction of the likely infecting organism(s), and to determine whether (surgical or
interventional) source control measures are indicated for definitive management of the
infective episode.
Physical examination may aid in establishing the origin of the infection, but radiologic
imaging studies are typically needed to document the precise site of origin and extent of
spread. A chest X-ray may support the diagnosis of pneumonia by showing lobar
consolidation or other acute alterations. If an effusion is also present, particularly in the
patient with pneumonia who has failed to respond to systemic antibiotics, the diagnosis of
empyema should be entertained, and investigated by aspirating a sample of the pleural
fluid.
Ultrasonography is inexpensive, and rapidly as well as widely available. Its greatest utility
lies in detecting infections arising from an obstructed abdominal hollow viscus – for
example, the gall bladder and biliary tree producing acute cholecystitis or cholangitis,
respectively, or from the urinary tract in the case of pyelonephritis. Ultrasonography is
operator-dependent, and its use may be limited by external dressings, or by significant
quantities of air in the gastrointestinal tract.
Computerised tomography (CT) scanning is the most useful diagnostic modality for patients
with deep space infections in the abdomen or thorax; it is also useful in evaluating the
extent of complex soft tissue infections. Oral or rectal contrast aids in CT interpretation by
delineating the lumen of the gastrointestinal tract, and by demonstrating leaks from the GI
tract when these are present. Intravenous contrast permits the identification of major
vascular structures, and can demonstrate areas of tissue non-perfusion, suggestive of
ischaemia or infarction. For more information see the PACT module on Clinical imaging
Accurate delineation of the anatomic site of infection provides a diagnosis of the infection
responsible for the septic state, and may facilitate image-guided management using
percutaneously placed drains, and therefore obviating the need for surgical intervention.
Fluid collections
suggestive of abcesses,
with drain in situ
Less common foci of infection should also be considered. In the patient with multiple
positive blood cultures, a diagnosis of endocarditis should be considered, particularly in the
patient with pre-existing risk factors – for example, valvular heart disease or a prosthetic
heart valve, intravenous drug abuse, a long-standing central venous catheter. The
diagnosis is supported by clinical features and established by transoesophageal
echocardiography and blood culture. Altered neurological status or focal findings suggests
the possibility of a cerebral or spinal focus, and should be assessed further by CT scan or
MRI. Invasive tests like bone marrow aspiration must be considered for systemic infections
such as tuberculosis especially in the immunocompromised host.
What is the infecting pathogen?
Accurate identification of the nature and antibiotic sensitivity profile of the infecting pathogen
enables the clinician to tailor antibiotic therapy, using a regimen that is effective, but narrow
spectrum, and so minimally disruptive to the patient's endogenous flora.
An initial prediction of the likely infecting pathogen is made on the basis of the presumptive
site of the infection (pneumonia, intra-abdominal infection, prosthetic device-related
infection, etc), and on the mode of acquisition – community-acquired or nosocomial.
Presumptive empiric therapy can then be initiated with reasonable confidence that the
actual infecting organism will be susceptible (see the PACT module on Severe
infection
).
Accurate identification of the infecting organism requires the results of microbiological
culture and sensitivity testing, which typically are not available for two or three days after the
initial specimens were taken. Once these data are available, the antibiotic spectrum should
be narrowed to target the particular pathogen specifically; if cultures are negative after 72
hours, strong consideration should be given to stopping all antibiotics. Alternately, other
causative organisms should be considered if the patient has persistent clinical signs of
sepsis.
Managing infection in the septic patient
Once a presumptive diagnosis of infection as the cause of sepsis is made, treatment should
be initiated expeditiously.
Systemic antibiotics appropriate to the working diagnosis and based on knowledge of local
resistance patterns, should be given as soon as cultures have been drawn and, ideally,
within an hour of presentation. Antibiotic selection is discussed further in the following
reference and in the PACT module on Severe infection
.
When a discrete focus of infection is identified, source control measures should be
considered, and implemented once the patient has been stabilised. Some general principles
underlie their use.
A 74-year-old man is admitted to the ICU following repair of a ruptured abdominal aortic
aneurysm; significant co-morbidities include hypertension, insulin-dependent diabetes mellitus, and
chronic obstructive pulmonary disease. He is intubated, and has a Foley catheter, pulmonary artery
catheter, peripheral intravenous, and nasogastric tube placed in the operating room (OR). What
specific sites of nosocomial infection is he at risk for? Why?
What organisms are most likely to be cultured from each of these sites?
Source control is a term that encompasses all those physical measures undertaken to
control a source of infection, to stop ongoing microbial contamination, and to restore optimal
anatomy and function. Source control measures are applicable to most infections causing
severe sepsis, and fall into two broad categories.
Drainage is the creation of a controlled sinus (a connection between a closed cavity and an
epithelial surface) or fistula (an abnormal communication between two epithelially-lined
surfaces). By draining an abscess (a localised focus of infection and host inflammatory
cells, walled off by a fibrin capsule), a deep-seated infection is converted to a controlled
sinus or a fistula (figure below).
Drainage creates a controlled sinus or
fistula
Draining abcesses,
percutaneously or
surgically
The drain is a foreign body
that keeps the tract open
Based on this principle, the optimal approach to achieving source control is that which
accomplishes the objective with the least degree of upset to the patient, and with full
consideration to minimising the risk associated with subsequent reconstructive efforts. For
example, perforated diverticulitis causing severe sepsis can be managed operatively by
resection of the involved segment of colon, followed by the creation of an end-colostomy, or
by primary anastomosis. Resection removes the site of ongoing contamination, while the
colostomy or anastomosis is the controlled fistula (because it is an abnormal
communication between two epithelially-lined surfaces). However, if the CT scan shows a
localised process, percutaneous image-guided drainage can accomplish the same
objectives with a lesser degree of physiologic insult to the patient. Drainage procedures are
most effective if the infected material is liquid; if it includes solid tissue, then debridement
must be employed.
Debridement is the mechanical removal of infected non-viable tissue, typically by surgical
resection, but in the case of superficial infection, by the use of dressing changes, or in the
case of an infected foreign body, by its removal. As a general rule, debridement of infected
necrotic tissue should be performed as rapidly as feasible: for patients with necrotising soft
tissue infection, for example, prognosis is directly related to the interval between onset and
definitive surgery. However, the benefits of debridement must be weighed against the risks
of intervention: for patients with infected peripancreatic necrosis, it has become evident that
delayed debridement is preferable, because of the substantial risk of uncontrollable
haemorrhage that may occur when adequate demarcation of viable and non-viable tissues
has not occurred.
The ultimate objective in managing the patient with severe sepsis or septic shock is to
achieve survival with a good quality of life. Careful consideration of the definitive
measures needed to manage a focus of infection can shorten the period of time the patient
is hospitalised, and improve the ultimate quality of life. For example, avoiding a stoma will
obviate the need for subsequent surgical reconstruction and its attendant risks. When a
stoma is deemed necessary, creating it in such a manner that it can be closed through a
single small incision will reduce the morbidity associated with subsequent reconstruction.
T HINK
For each of the following clinical scenarios resulting in severe sepsis or septic shock,
consider:
 What are the options for achieving source control?
 How can the problem be managed using the principles outlined above?
 What is your preferred approach and why?
A 66-year-old woman presents to the emergency department with fever, chills,
jaundice, and right upper quadrant pain, 14 years after a cholecystectomy. Her blood
pressure is 70/40, and her heart rate 130/minute. An ultrasound exam shows the
common bile duct to be dilated to 1.8 mm.
A 47-year-old man presents with diffuse abdominal pain; he is hypotensive and
tachypnoeic, with a temperature of 38.7 °C. Following resuscitation, he undergoes CT
scanning which shows sigmoid diverticulitis with an associated abscess.
An 86-year-old woman develops confusion, a supraventricular dysrhythmia, and
oliguria, three days following a right hemicolectomy for cancer. She is afebrile. Her
CT scan is shown below.
CT scan with free air in the
abdomen suggestive of
leaking anastomosis
A 59-year-old man, intubated five days after a right lower lobectomy for lung cancer,
develops a fever, and a new right-sided infiltrate. There is an associated pleural
effusion; diagnostic thoracentesis yields Gram-positive cocci.
T HINK







Invasive devices are a common nidus of infection in critically ill patients. What factors
contribute to device colonisation?
What steps can be taken to reduce the risk of colonisation?
For which of the following nosocomial infections should device removal be an integral
part of infection management, and under what circumstances?
Bacteraemia
Urinary tract infection
Ventilator-associated pneumonia
Septic thrombophlebitis
Peritonitis during chronic ambulatory peritoneal dialysis
A-V or V-V shunts for chronic renal replacement therapy
Indwelling central catheters (short or long-term)
Non-infective causes of severe SIRS
A diagnosis of severe sepsis or septic shock implies an infective aetiology. However an
identical clinical syndrome may evolve in the absence of infection, and management
principles do not differ, with the important exception that anti-infective measures are
unnecessary, and potentially even harmful.
To establish that infection is not the cause of the acute illness requires primarily the
exclusion of infection as a cause, and secondarily making a reasonable alternative
diagnosis (table below). Appropriate empiric antibiotics and a systematic attempt to
establish an anatomic and microbiologic diagnosis are indicated if there is a moderate or
high probability of an infective cause. However, it is equally important, if these investigations
fail to confirm a diagnosis of infection, that empiric antibiotics be stopped as soon as the
diagnosis has been ruled out, and certainly by 72 hours after initial presentation.
Injury
Ischemia -reperfusion
Inflammation sterile
Immune
Idiopathic
Indicrine
Intoxication
Iatrogenic
4/ ADJUNCTIVE THERAPY FOR SEPSIS
Multiple, Trauma, VILI, Hge
Limb ischemia, opening the abd in
compartmental syndrome,
rupture Aorta aneurysm
Acute pancreatitis
SLE, GVHt, Transplant rejection,
collagenic
Toxic epidermal , wegners, TTP
DKA, Addisons, Hyperthyrodism
ASA
Transfusion , drugs
Despite advances in our understanding of sepsis, the mortality after treatment is over 50%
in the presence of shock. Earlier in this module, the importance of diagnosis, early
resuscitation and directed therapy for sepsis were emphasised. It is, however, crucial to
appreciate that several other aspects of the disease and its treatment also impact on the
outcome from sepsis. This discussion will highlight the adjunctive interventions, based on
available evidence, which may be considered part of a comprehensive approach to
treatment. Such interventions must equally be with minimal risk of adverse events. The
considerations for paediatric patients are uniquely different and will not be addressed.
Several therapeutic interventions have been investigated for sepsis in an
attempt to reduce morbidity and mortality. Where the benefits of such therapy
are unproven, their introduction cannot be justified.
Adjunctive therapy fo
*rhAPC: recombinant h
activated protein C,
DVT: deep vein thromb
LOS: length of stay,
NOS: Nitric oxide synth
Intervention
Rh APC
Steroids
Glucose control
Blood products
Sedation ,
analgesia, ms
relaxants
DVT prophylaxis
Stress ulcer
prophylaxsis
Endpoint
Dose related antiinflammatory, anticoagulant,
profibrinolytic
Dose related reduction NOS
Normoglycemia
Increase oxygen delivery
Reduce O2 consumption
Prvent stress ulcer
SE
Bleeding
Hypoglycemia
Infection
Increase
length of stay,
polyneropathy
Bleeding
Increase
nosocomial
pneumonia
Recombinant human activated protein C
Many novel therapeutic agents have been investigated over the last three decades. None have
shown incontrovertible evidence of benefit in patients with sepsis. Recently, recombinant human
activated protein C (rhAPC) has been shown to reduce mortality (absolute mortality reduction of
6.1%) in adult patients with severe sepsis when administered to patients within 24 hours of
diagnosis of severe sepsis and sepsis induced organ dysfunction. There are many postulated
mechanisms of action including reduction of thrombin formation, an anti-inflammatory effect
and reduction of rolling of monocytes and neutrophils. The dose required is 24 µg/kg/hour for 96
hours. Bleeding is the commonest complication associated with therapy. Treatment with rhAPC
should be avoided in patients with active bleeding or at high risk for bleeding and the drug has
not yet been shown to be effective in less severe septic patients.
Treatment must be curtailed where significant risks are associated with drug
administration.
The Surviving Sepsis Campaign guidelines (2008) recommend that the drug be considered in
septic patients with an APACHE 2 score ≥25 but not in those with a score <20.
N OTE
The European Medicines Agency (EMeA) and other regulatory bodies have
individualised the indications for rhAPC use in severe sepsis and it is advised that you
know the licensed indication in your own jurisdiction.
http://www.emea.europa.eu/htms/human/epar/a.htm
In patients who require surgery, the drug should be discontinued for at least one hour
prior to surgery. It may be recommenced one hour after minor procedures and 12 hours
after major surgery – once concern re bleeding complications has subsided.
Absolute contraindications for rhAPC







Active internal bleeding
Recent (within 3 months) haemorrhagic stroke
Recent (within 2 months) intracranial or intraspinal surgery, or severe head
trauma
Trauma with an increased risk of life-threatening bleeding
Presence of an epidural catheter
Intracranial neoplasm or mass lesion or evidence of cerebral herniation
Thrombocytopenia <30 x 109/l.
Steroids
Earlier studies using high dose steroid therapy for sepsis demonstrated no mortality benefit
and were associated with an increased prevalence of nosocomial sepsis. Low dose
hydrocortisone (200-300 mg per day) was shown in the Annane (single-centre) study to
reduce mortality in corticotropin unresponsive patients with septic shock who require fluids
and vasopressor support. However, the larger, multi-centre CORTICUS study showed only
that hydrocortisone hastened the reversal of shock in those in whom shock was reversed.
Hydrocortisone did not improve survival or reversal of shock in patients with septic shock,
either overall or in patients who did not have a response to corticotropin.
When using hydrocortisone, treatment should not be delayed for confirmation of the
presence of hypoadrenalism (by corticotropin stimulation testing).
The mechanism of action is thought to be via improving catecholamine receptor
responsiveness to exogenous catecholamines. Modulating immune responsiveness in
patients who have relative adrenal insufficiency is also thought to occur. Treatment with
steroids should be weaned once shock has resolved. It is common practice to administer
steroids for 5-7 days as abrupt withdrawal of therapy may result in adverse haemodynamic
and immunological effects. There are no data supporting the use of steroids for protracted
periods and a higher rate of steroid-related septic complications have been confirmed, even
in the 11 day duration of therapy utilised in the CORTICUS study (above).
N OTE
T HINK
Therapeutic interventions must only be introduced when there is clear evidence
of benefit AND where the risks of treatment are out-weighed by the benefits.
A known asthmatic is being ventilated for severe bronchospasm. He develops septic
shock from a nosocomial pneumonia. What are the considerations for steroid therapy
in this patient? The following scenarios illustrate the multiple reasons to start steroids
in this, and many other cases.
Three scenarios are possible in this case:
 Steroids prior to admission: If the patient is a known asthmatic, it is
possible that he would have been receiving steroids prior to ICU
admission. In this instance, he would be on steroids at the time of
development of nosocomial sepsis, in which case steroid therapy must
be continued to treat acute on chronic bronchospasm and must be
continued after the episode of septic shock has resolved.
 Steroids for bronchospasm: The patient may not have been on steroids
prior to admission in which case steroids may have been administered
for the management of refractory bronchospasm. In this instance, steroid
therapy must be withdrawn after resolution of bronchospasm or seven
days after treatment for septic shock, whichever is later.
 Steroids for septic shock: The standard approach to steroid therapy for
fluid resuscitated and vasopressor-dependent septic shock should be
considered.
A patient has been treated for an episode of septic shock with steroids. Two days later, he
develops another episode of septic shock. Would you administer steroids again? Justify your
answer.
Glucose control
Glucose control was demonstrated to reduce morbidity and mortality in surgical patients
when the serum glucose was kept between 4.4-5.5 mmol/l (80-100 mg/dl). In medical
patients there is a significant reduction in morbidity, however, the benefits were more
significantly realised in medical patients with an ICU stay >3 days. Glucose control should
be initiated as soon as initial stabilisation of the patient has been completed. It is
recommended that the glucose level to be targeted is <8.3 mmol/l (<150 mg/dl), with a goal
of 4.4-5.5 mmol/l (80 to 100 mg/dl). This higher level is suggested because of the significant
risk of hypoglycaemia. Glycogen depletion during sepsis increases the risk of
hypoglycaemia. Hypoglycaemia has been reported even during the conduct of rigorous
clinical trials. This is a serious life-threatening complication in the sedated patient and must
be assiduously avoided.
Clear adherence to a well thought through protocol, well-trained nursing/medical
involvement and supervision, regular blood testing and routine administration of glucose
containing solutions (to avoid inadvertent hypoglycaemia) must be considered mandatory
when patients are receiving insulin. Hourly glucose testing should be performed until the
levels have stabilised and thereafter four hourly testing is acceptable. Point-of-care testing
has been shown to overestimate glucose levels and must be used with caution.
T HINK
What are the clinical signs of hypoglycaemia? How would you suspect and diagnose
hypoglycaemia in a sedated, ventilated patient?
Blood products
Red blood cells
Current data suggest that a haemoglobin level between 7-9 g/dl is as effective as a level
between 10-12 g/dl in resuscitated patients. The Rivers trial mentioned above indicates this
may not be true in sepsis patients with shock and persistent lactic acidosis or low ScvO 2 on
initial presentation. Red blood cells (RBCs) increase the oxygen carrying capacity of blood
but do not immediately release bound oxygen and therefore may not result in increases in
oxygen consumption. In addition, increasing the RBC concentration may adversely affect
blood rheology thereby reducing oxygen delivery. Since the ideal haemoglobin
concentration is unknown, RBC transfusion should likely be directed to achieving a level of
7-9 g/dl. In certain groups of patients where oxygen delivery and consumption is critical e.g.
post-cardiac surgery or persistent ischaemic lactic acidosis, higher levels of haemoglobin
(10-12 g/dl) may be preferable.
Platelets and fresh frozen plasma
Platelets should be administered when the count is <5 x 109/l since these patients are at
risk for spontaneous bleeding. A platelet count >50 x 109/l is recommended prior to surgery.
Abnormal laboratory tests of coagulation alone should not be used as an indication for
coagulation factors. Fresh frozen plasma and/or platelets should be used in the presence of
documented bleeding or prior to surgery and abnormal laboratory tests suggest their use.
For more information see the PACT module on Bleeding and thrombosis
Erythropoietin and antithrombin
There are no data to support the use of either therapy in patients with sepsis. Erythropoietin
may be indicated for other conditions e.g. pre-existing chronic renal failure and impaired
erythropoietin production. High dose antithrombin increases bleeding when co-administered
with heparin.
T HINK
A patient with intra-abdominal sepsis receiving rhAPC requires major surgery. The drug
infusion is stopped one hour prior to surgery. Excessive blood loss with anaemia occurs in ICU
after surgery. What causes of bleeding should be considered and what should be done?
Analgesia, sedation and muscle relaxation
Patients with sepsis frequently require mechanical ventilation. Details on airway care,
intubation and extubation, ventilation, analgesia, sedation and the use of muscle relaxants
are covered in the PACT modules on Airway management
, Mechanical
ventilation
and Sedation
.
T HINK
The ICU nurse reports that a patient with sepsis requires increasing doses of sedative
drugs without achieving adequate sedation. What are the likely causes for this
behaviour?
Thromboembolism prophylaxis
ICU patients are at risk for the development of deep vein thrombosis (DVT) and pulmonary
embolus (PE). Prophylaxis has been shown to reduce the prevalence of these
complications, which should therefore be considered to be standard care for patients with
sepsis. Unfractionated heparin (UFH) (eight hourly) is as efficacious as low molecular
weight heparin (LMWH) (once daily) when administered subcutaneously. UFH should be
used instead of LMWH when cost constraints are a factor. LMWH is renally excreted and
may accumulate in patients with severe renal dysfunction. A dosage reduction/monitoring of
anti-xa has been recommended in patients with creatinine clearance of <30 ml/minute.
In patients with active bleeding, coagulation disturbances and proven heparin-induced
thrombocytopenia (HIT), both agents should be avoided and anticoagulation with nonheparin agents may be considered. Mechanical devices should be used in such patients.
Patients at high risk for DVT and PE should receive both drug prophylaxis (UFH or LMWH)
and mechanical devices (compression stockings or intermittent compression devices).
T HINK
What factors constitute high risk for DVT/PE? What are the clinical signs of PE?
For more information see the PACT module on Bleeding and thrombosis
.
Stress ulcer prophylaxis
Upper gastrointestinal bleeds occur less frequently in ICU patients who receive prophylaxis
for stress ulcers but there is no evidence of a reduction in mortality from this therapy. Given
that mechanical ventilation, coagulopathy and hypotension are the risk factors and that
these occur commonly in patients with sepsis, prophylaxis should be administered in all
patients with severe sepsis. Histamine-2 (H2) receptor antagonists and proton pump
inhibitors are equivalent in producing this benefit. The choice of agent should be based on
cost, availability of drug and side effects associated with these agents. Sucralfate has been
shown to be less effective than H2 receptor antagonists. For more information see the
PACT modules on Nutrition
and Infection control strategies
.
Intravenous immunoglobulin
Several studies have suggested the use of intravenous immunoglobulin (IVIG) as an
adjunctive intervention in patients with sepsis. The rationale is based on augmentation of
the immune response to infection. The Surviving Sepsis Campaign Guidelines suggest that
immunoglobulin may be considered in children with severe sepsis but there is no
corresponding suggestion of recommendation in adults. There have been three recent
meta-analyses but given differences in study methodology, issues relating to dose
standardisation and safety, not to mention other advances in the management of sepsis, a
large, prospective randomised trial is indicated before this form of treatment should be
considered as routine adjunctive therapy.
Granulocyte colony-stimulating factor
Granulocyte colony-stimulating factor (G-CSF) has been used in neutropenic patients for
several years to bolster receptor-mediated neutrophil and macrophage responsiveness and
has been used clinically in severely septic, neutropenic neonates. Serum concentrations of
G-CSF are elevated in sepsis but have no relationship to outcome. The Surviving Sepsis
Campaign Guidelines do not include mention of G-CSF and further studies are required to
elucidate where this intervention may be beneficial.
In conclusion, adjunctive therapeutic strategies have reduced morbidity and mortality
associated with sepsis and should be routinely considered in clinical practice. These
interventions should be carefully evaluated for benefit to ensure that unacceptable side
effects (e.g. hypoglycaemia, bleeding or superinfection are controlled as far as possible. An
area that will require careful ongoing consideration is the potential negative consequences
from the multiple therapies that have been advocated for seps
5/ MINIMISING ORGAN DYSFUNCTION IN ICU
Acute multiple organ dysfunction is the defining complication of severe sepsis and septic
shock. Organ dysfunction arises as a consequence of the profound systemic homeostatic
derangements that accompany an uncontrolled systemic inflammatory response. It also
develops, however, as a consequence of what the healthcare team does to treat these
derangements. Organ dysfunction therefore may develop because of both the successes
and the shortcomings of contemporary ICU care. Whilst all organ systems are affected, the
cardiac and respiratory dysfunction typically manifest early in the clinical course of severe
sepsis.
What is MODS?
MODS can be defined as the development of acute physiologic organ system dysfunction
following an acute life-threatening insult. Its manifestations may involve virtually every
aspect of normal physiologic function but by convention, the syndrome of MODS is usually
considered to involve derangements in the function of six organ systems – the respiratory,
cardiovascular, renal, haematologic, hepatic, and neurologic systems. The concept that the
syndrome represents dysfunction rather than failure is preferred, since the process is
reversible, and prognosis correlates not only with the number of failing systems, but also
with the degree of dysfunction within a system. This conceptual model is reflected in the
emergence of a number of similar systems to quantify the severity of the syndrome using a
score (Table below).
Measuring
Organ
Dysfunction:
the MOD and
sequential
organ failure
assessment
(SOFA) scores
The capacity to quantify MODS provides the clinician with a tool to measure changes in
global severity of illness over time, and the investigator with a means of evaluating the
epidemiology and natural history of critical illness. The MODS score uses six continuous
physiologic variables, and so measures organ dysfunction independent of a therapeutic
decision, since such decisions vary and may not only reflect evolving illness, but contribute
to its evolution. Function of the cardiovascular system is measured using a simple
calculated variable, analogous to the PO2/FiO2 ratio – the pressure-adjusted rate (PAR),
calculated as the product of the heart rate and CVP, divided by the mean arterial pressure,
and reflecting hypotension that does not respond to fluid resuscitation:
A normal value is 10 or less; increasing values reflect worsening cardiovascular function. In
the absence of an actual measure of right atrial pressure, a normal value of 8 is used. The
SOFA score incorporates therapy-dependent variables in the measurement of respiratory
and cardiovascular dysfunction; urine output is also included as a measure of renal function.
These differences, however, have only a minimal effect on the reliability of each.
The measuring systems provide different and complementary information on the clinical
course of the patient. The score on the day of admission is a measure of illness severity at
the onset of care, and correlates directly with ultimate prognosis: it is, in effect, a snapshot
of how sick the patient is before critical care begins. Scores measured on a daily basis
provide a reflection of how the global course of illness is evolving. Moreover the domains of
improvement or deterioration can be followed over time, particularly if the raw data for each
variable are used – is respiratory function, measured by the PaO2/FiO2 ratio, getting better
or worse? Is there particular deterioration or improvement in one organ system that might
point to a correctable problem? Change over time can also be measured using the delta
score – either by taking the worst values in each system over the ICU stay, and subtracting
them from the admission score, or by looking at the change in scores from one day to the
next; the former calculation provides a global measure of new organ dysfunction developing
during the ICU stay, and so is a reflection of potentially preventable ICU morbidity.
Preventing organ dysfunction in the critically ill
The ultimate objective of critical care practice is to support a patient during a period of lifethreatening physiologic instability, while treating the process that triggered the illness on the
one hand and minimising the associated multi-organ dysfunction including the adverse
consequences of support measures. For convenience, this topic can be addressed in terms
of one organ at a time.
Respiratory dysfunction
The acute lung injury (ALI) and acute respiratory distress syndrome (ARDS) reflect a
particularly severe form of the respiratory dysfunction that develops in many critically ill
patients. This respiratory dysfunction results from both the systemic inflammatory response
to the acute injury that led to ICU admission, and the local response of the lung to
additional, and preventable insults arising from the technique of ventilatory support used, or
superimposed lung infection producing ventilator-associated pneumonia. A state of
generalised vasodilatation and increased capillary permeability results in interstitial oedema
in the lung, and exudation of protein-rich fluid into the alveolar space. Both of these
compromise gas exchange, producing alterations that can be detected early in the course of
the process as tachypnoea, hypoxaemia, and hypocarbia, as well as loss of lung
compliance. The institution of mechanical ventilation introduces a new insult to the
vulnerable lung, as positive pressure ventilation applies unnatural shear forces to the lung
units, and repeated opening and closing of these units causes local tissue injury.
This injury evokes a further inflammatory response, with the influx of neutrophils into the
lung, further exacerbating the local injury, and compromising gas exchange. Thrombosis in
small, medium, or large vessels – secondary either to systemic activation of the coagulation
cascade, or to embolisation from the extremities, further compromises gas exchange.
Aspiration of bacteria from the stomach, oropharynx, and endotracheal tube evokes a
further response, and adds to the complex picture of ALI. Finally, the process of tissue
repair, with local fibrosis, contains the injured tissue, but at the cost of its function, and gas
exchange is even further impaired. While the initial insult has usually already occurred by
the time the physician sees the patient, the subsequent secondary injury is eminently
amenable to preventive measures.
Gas exchange in the lung is compromised by alveolar oedema, and so early pulmonary
function can be improved by judicious fluid resuscitation, administering a volume adequate
to support the circulatory system without producing excessive alveolar or interstitial
oedema. Monitoring resuscitation through the measurement of central venous pressure can
aid in attaining this objective. It remains controversial whether clinically relevant oedema
can be further reduced by the use of colloids such as albumin or starch solution in
preference to crystalloids.
The development of ventilator-associated pneumonia (VAP) exacerbates acute respiratory
dysfunction. A variety of strategies have been shown to be effective in reducing the risk of
VAP, including elevation of the head of the bed, use of closed system suction devices,
reduced frequency of ventilator tubing changes, and subglottic drainage of secretions.
Selective decontamination of the digestive tract (SDD) – a strategy that reduces pathologic
colonisation of the oropharynx and GI tract through the use of topical non-absorbed
antibiotics and a short course of a parenteral agent – has been shown to be very effective in
reducing the risk of VAP. Similarly, avoiding over-sedation, and removing the endotracheal
tube as early as is feasible are additional effective strategies to lower the risk of VAP.
Protective lung ventilation strategies, accomplished by the use of low tidal volumes (68 ml/kg) and higher levels of PEEP, have been shown to reduce not only pulmonary, but
also distant organ injury, and to improve survival. These effects may be modulated by the
gradual introduction of pressure control ventilation and appropriate dosing of sedation. For
more information see the PACT modules on Mechanical ventilation
, Respiratory
failure
, Respiratory monitoring
and Haemodynamic monitoring
.
Cardiovascular dysfunction
The cardiovascular dysfunction of MODS includes peripheral vasodilatation, increased
capillary permeability, microvascular occlusion with deranged capillary flow, and myocardial
depression. While these abnormalities are not readily manipulated directly, the ultimate
consequence – tissue hypoxia – can be attenuated by judicious clinical management.
The adverse effects of tissue oedema can be minimised. The use of a goal-directed
approach, dosing fluid volumes to physiologic response can aid in refining fluid
resuscitation. Commonly, once resuscitation has been accomplished, the patient will remain
in a significantly positive fluid balance for many more days; cautious removal of this excess
volume using pharmacologically-induced diuresis or renal replacement therapies can aid in
mobilising this fluid.
The objective of haemodynamic support is to maximise oxygen delivery to the tissues
(DO2). Since tissue oxygenation cannot be readily measured directly, it must be inferred
indirectly. A reduced venous saturation indicates increased O2 extraction, and so suggests
tissue hypoxia. Similarly an elevated lactate level suggests anaerobic metabolism
secondary to reduced DO2. Both of these abnormalities have multiple causes, and therefore
must be interpreted carefully. Hypotension is also inferred to reflect impaired DO 2 since one
must assume either inadequate cardiac output or loss of vascular tone, or both. However
increasing blood pressure pharmacologically may not increase DO2, but actually reduce it,
since increased pressure is achieved through constriction of, and perhaps reduction of flow
in, small vessels. Other effects of circulating mediators may decrease the red cell ability to
deform and flow through the capillaries, consumption of clotting products may cause microocclusion and all of these types of events may actually promote a further shunting of
oxygenated blood, therefore an increase in ScvO2 in the face of increasing clinical
dysfunction and lactic acidosis. For more information see the PACT module on
Hypotension
.
Optimal cardiovascular management to minimise new organ dysfunction is complex, often
counter-intuitive, and not at all well-defined. Beyond adequate resuscitation as outlined in
our second task
, there is no evidence that invasive monitoring with a pulmonary artery
catheter, attempts to drive haemodynamic parameters to supranormal values, or aggressive
transfusion of RBCs can improve outcome.
Renal dysfunction
Renal dysfunction in the critically ill patient most commonly results from hypoperfusion, and
so early, aggressive, and adequate resuscitation is the key to minimising subsequent renal
dysfunction. In the patient with intra-abdominal pathology – for example, severe necrotising
pancreatitis, a ruptured abdominal aortic aneurysm, or multiple abdominal trauma – renal
perfusion may be impaired following resuscitation as a consequence of the development of
an abdominal compartment syndrome. The diagnosis is established by the measurement of
bladder pressure as a reflection of intra-abdominal pressure. Finally, nephrotoxic
medications can contribute to the evolution of acute tubular dysfunction and acute kidney
injury (AKI). For more information see the PACT modules on Acute renal failure
and
Abdominal problems
Hepatic dysfunction
The causes of evolving hepatic dysfunction in the critically ill are multifactorial, however
several modifiable causes deserve particular attention. Ischaemia – evidenced as
biochemical evidence of hepatocellular and reflected in significant rises in hepatic
transaminases – is the most common cause of early hepatic dysfunction; hyperbilirubinemia
is usually absent, or of very modest degree. Total parenteral nutrition can induce
hepatocellular cholestasis; enteral nutrition is preferred if possible. Massive transfusion may
cause jaundice. Finally, hepatotoxic medications may produce a picture of evolving
cholestasis. Once liver failure has occurred, the lactate levels will become even less specific
to tissue perfusion. For more information see the PACT module on Acute hepatic failure
Haematologic dysfunction
Early haematologic dysfunction in the critically ill may be caused by depletion of coagulation
factors as a consequence of massive bleeding. Alternatively, such dysfunction may occur
as a result of intravascular consumption during the process of disseminated intravascular
coagulation (DIC). Special attention should be paid to the inverse relationships between
consumptive coagulopathy and tissue hypoperfusion. In both cases, prophylaxis is targeted
at the cause. Thrombocytopenia may also reflect the development of heparin-induced
thrombocytopenia (HIT). Discontinuation of heparin exposure is therapeutic. For more
information see the PACT module on Bleeding and thrombosis
Neurologic dysfunction
Although abnormalities in peripheral nerve function have been described in the critically ill,
the most obvious and clinically most important manifestation of altered neurologic function
is a reduced level of consciousness. Excessive sedation is an important contributing factor,
and the ICU stay can be shortened, and outcomes improved, by a strategy of deliberate
daily wakening of the critically ill patient. Drowsiness, agitation, or confusion may also
reflect the side effects of other medications, the sequelae of aggressive fluid resuscitation
with subclinical cerebral oedema, or the effects of disruption of normal patterns of sleep and
day/night cycling.
While corticosteroids may improve haemodynamic function early in the course of septic
shock, their use has been associated with muscle weakness during convalescence from
critical illness. For more information see the PACT module on Neuromuscular
conditions
Nosocomial infection contributing to organ dysfunction
Optimal management of the critically ill patient necessitates a careful titration of support to
maximise benefit and minimise harm.
Nosocomial infection in the critically ill may arise from exogenous reservoirs, but more
typically results from infection with endogenous organisms whose emergence reflects a
combination of altered patterns of colonisation, and impairment of physical and other host
defence barriers.
Invasive devices such as central catheters, endotracheal tubes, and urinary catheters are
particularly susceptible to colonisation by micro-organisms that produce biofilms e.g.
coagulase-negative staphylococci. The colonised device both bypasses normal physical
barriers, and serves as a reservoir for continuous microbial seeding. Successful treatment
involves removal of the device when feasible; prophylactic measures include adequate
sterile technique during device insertion, and reducing unnecessary usage.
The predominance of antibiotic-resistant organisms in nosocomial ICU-acquired infection
also reflects the important role played by antibiotic pressures in selecting for resistant
species. Judicious use of antimicrobial agents based on the use of narrow spectrum,
culture-guided therapy, shorter duration of treatment, and discontinuation of empiric
antibiotics in the face of negative cultures are all important strategies to reduce antimicrobial
pressures on microbial ecology.
Randomised controlled trials and systematic syntheses of the data from these provide
strong evidence that both rates of nosocomial infection, and ICU mortality can be reduced
through the use of SDD. SDD entails the topical application of non-absorbed antibiotics with
broad spectrum activity against Gram-negative aerobes (polymyxin B and tobramycin) and
fungi (amphotericin), supplemented by a 3 to 4 day course of a systemic agent such as
cefotaxime. The regimen preserves the anaerobic flora of the gastrointestinal tract, and has
no significant activity against Gram-positive organisms. Despite extensive evidence of
efficacy, and despite the lack of any compelling evidence of harm secondary to the
emergence of resistant organisms, SDD is not widely used
Treating organ dysfunction in the critically ill
A comprehensive overview of the support of the critically ill patient with organ dysfunction
would comprise a textbook of critical care. Specific considerations in the support of
individual failing organs are covered in other modules of PACT. For our purposes here, we
will focus on a few basic principles in the provision of such support.
Organ system support does not cure the underlying disease: it merely sustains life while the
clinician identifies a treatable process, or sometimes, while the patient gets better despite
the absence of a compelling diagnosis. The degree of organ system support is a reflection
of the gravity of the clinical situation, and mortality increases with both the number of failing
organ systems, and the degree of dysfunction within each. The goal of therapy is to liberate
the patient from exogenous support.
With this objective in mind, it is important to acknowledge that patients do not require the
support we provide; rather we choose (for clinically compelling reasons) to provide that
support, and equally, good clinical care entails careful consideration of how important that
support is to a successful outcome. If the mean arterial pressure is 60, therefore, it is
important to ask the question, 'Do I need to increase it to improve tissue oxygenation?',
rather than simply to institute vasoactive therapy to increase it to an arbitrary level such as
65 or 70. The patient has no intrinsic need for vasopressors; rather you have a therapeutic
decision to make, whether artificially increasing the mean arterial pressure will improve
tissue oxygenation, and so increase the probability of survival. Often, as in this case, there
are little data from clinical trials to guide that decision: good clinical judgment and careful
evaluation of the consequences of a decision must guide practice.
Similarly, patient care is best served by an explicit attempt to liberate patients from the need
for exogenous support as rapidly as is safe and feasible. While there may be something
comforting about a sedated, haemodynamically stable patient receiving full mechanical
ventilation, explicit attempts to wean the patient by daily spontaneous breathing trials are
associated with a superior clinical outcome.
Finally, organ dysfunction is the clinical consequence of a pathologic process, and faced
with new or evolving organ dysfunction, every effort should be made to identify a treatable
cause. Does new onset organ dysfunction herald an unrecognised complication of recent
surgery, or an adverse effect of a newly prescribed medication? Does it reflect the adverse
sequelae of another intervention such as mechanical ventilation, transfusion, or total
parenteral nutrition? Is it the initial presentation of a nosocomial infection? The desired
clinical trajectory for the critically ill patient is one of improving physiologic function and
liberation from exogenous support. Deviations from this course must be managed, but more
importantly, they must be viewed as possible clinical evidence of a new complication.
Further information on organ dysfunction and support can be found in PACT modules on
Haemodynamic monitoring
, Acute renal failure
, Oliguria and anuria
, Acute
hepatic failure
, Bleeding and thrombosis
, Severe infection
Organ dysfunction and clinical futility
Although increasing degrees of organ dysfunction portend a worse clinical outcome, organ
dysfunction alone is not the factor that guides a clinician to suggest termination of
aggressive support. Organ dysfunction is a dynamic process and the critical factor in
determining its reversibility is its cause and its evolution over time. The prognosis for
ultimate survival is reduced when organ dysfunction fails to resolve over an extended period
of time, or when organ dysfunction worsens in the absence of a treatable cause. However
decisions regarding the appropriateness of continuing life sustaining therapy are
individualised and grounded in considerations of the presence of treatable disease, the
influence of co-morbid conditions, and patient preferences for life prolonging support.
Consideration of the degree or trajectory of organ dysfunction can support, but not
determine these decisions.
CONCLUSION
Sepsis and SIRS are common causes of critical illness, MODS and of morbidity and
mortality. The key to a successful approach to clinical management is timely recognition of
sepsis/SIRS, early resuscitative treatment, concurrent diagnosis and specific therapy
including surgical/interventional source control where indicated. Adjunctive therapies can
improve the clinical course and survival. Organ supportive therapy is the mainstay of critical
care but needs to be titrated carefully to achieve maximum benefit while minimising any
associated adverse effects. Multidisciplinary collaboration is important to clinical
management.
PATIENT CHALLENGES
A 67-year-old male presents with a two-day history of chills, high fever, cough and
shortness of breath. He is known to have diabetes mellitus type II, mild hypertension and
mild COPD. On admission to the emergency department (ED) the patient appears ill, is
coughing, restless and sleepy, but rousable. Body temperature is 40.2 °C, blood pressure
80/40 mmHg, heart rate 120 b/min. (regular), respiratory rate 26/min.
On physical examination crackles are present over the right hemithorax
and a stat chest X-ray (figure) shows a consolidation mainly in the right
middle and lower lobes.
N OTE
Recognition of the life-threatening nature of this condition and the
need of prompt adequate treatment is mandatory. It may save his
life.
Laboratory results show:











A metabolic acidosis (pH 7.21, HCO3- 16 mmol/l)
Hypoxaemia (PaO2 58 mmHg [7.7 kPa], pCO2 25 mmHg [3.3 kPa], SaO2 89% on
2 l O2 by nasal cannula)
Haemoglobin 6.8 mmol/l (10.9 g/dl)
Leukocytes 16.4 x 109/l (80% granulocytes)
Platelets 60 x 109/l
Creatinine 160 µmol/l (1.8 mg/dl)
Urea 14 mmol/l (39.2 mg/dl)
Bilirubin 10 µmol/l (0.58 mg/dl)
Liver enzymes slightly elevated
Lactate 4.3 mmol/l (38.7 mg/dl)
Glucose 14 mmol/l (252.3 mg/dl)
What is the diagnosis? How urgent is treatment?
Learning issues
Recognition of severe sepsis and septic shock
What is your initial treatment?
Refresh your knowledge on treatment modalities of septic shock. This will guide the choice of your
therapy.
You are pursuing a two-pronged approach to the severe sepsis/septic shock involving
optimisation of tissue perfusion and specific treatment of the cause of the infection/sepsis.
When cultures have provided the identification of the causative organism(s) and their
sensitivity/resistance pattern, antibiotic therapy can be adjusted to these findings. Septic
shock, with its concurrent generalised tissue hypoperfusion, is the major cause of MODS in
the critical care situation. At this time point there is already dysfunction of several organ
systems.
Learning issues
Identification of clinical situations prone to develop MODS
Strategies to prevent MODS or further aggravation of organ dysfunction
Specify some of these organ dysfunctions.
Is rapid attention to reversing tissue hypoxia/hypoperfusion important to the final outcome?
Learning issues
Reversing tissue hypoxia/hypoperfusion
How can unintended, additional deterioration of organ dysfunction in your patient be
prevented?
N OTE
Be constantly aware of the possibility of iatrogenic injury and the risk of nosocomial
infection.
After initial resuscitation in the ED, the patient is admitted to the ICU. In the following hours
the patient becomes haemodynamically stable after fluid treatment and institution of
norepinephrine (0.4 µg/kg/min). Blood oxygenation, however, further deteriorates in the
hours following admission and the patient is intubated and mechanically ventilated (TV 6
ml/kg, PEEP 8 cmH2O, FiO2 0.5) with improvement in blood gases (PaO2 70 mmHg [9.3
kPa], SaO2 95%).
An initial Gram stain of a tracheal aspirate shows an abundance of Gram-positive diplococci
and cultures of both sputum and blood confirm two days later the presence of penicillinsensitive pneumococci. The initially instituted broad-spectrum antibiotic regimen is
narrowed down to high-dose penicillin i.v. on day two.
Learning issues
Management of MODS in septic patients
N OTE
All septic shock patients should be treated in ICU where possible.
Learning issues
Using laboratory data to narrow the antimicrobial spectrum of therapy
PACT module on Severe infection
How do you manage the hyperglycaemia in your patient?
Is there evidential support for tight glucose control in this clinical scenario? Are there cautions
required?
Learning issues
Blood glucose control
On the day of admission, insertion of a pulmonary artery catheter is considered but at this
time not done since after resuscitation, the patient remains haemodynamically stable and
starts to diurese (40 ml/hour). Moreover, metabolic acidosis gradually improves and 12
hours after the start of treatment the blood lactate level has fallen to 2.8 mmol/l. Over the
next two days the patient further improves, temperature declines and there is a slight
increase in PaO2/FiO2 ratio. Also, renal clearance improves as evidenced by a decreasing
serum creatinine and platelet count increases to 85 x 109/l.
What do you do now to minimise the risk of iatrogenic exacerbation of organ
dysfunction?
N OTE
Insertion of a
pulmonary
artery catheter
or other
invasive
monitoring
devices are not
routine
procedures in
septic shock.
They may be
useful options
however, when
haemodynamic
instability
persists with
an adequate
CVP, or when
ARDS and
renal
impairment
develops.
Careful
clinical
observation
and
monitoring of
intervention
targets are the
cornerstones
of
management.
PACT module on Haemodynamic monitoring
On day four after admission, however, the patient becomes haemodynamically unstable
and the vasopressor dose has to be increased (dopamine 15 µg/kg/min, norepinephrine 0.6
µg/kg/min with a CVP of 13 mmHg) to attain a MAP of 70 mmHg . Body temperature rises
again above 38.5 °C, and shows peaks of 40 °C, platelet count falls to 45 x 109/l and
oliguria worsens.
Apparently something is wrong with your patient now. What is your strategy?
Think logically and
systematically
Causes of haemodynamic instability in septic patients with MODS
What do you consider a likely cause of the haemodynamic instability in this patient and how do
you evaluate for this?
N OTE
Several causes can be simultaneously operative
PACT module on Haemodynamic monitoring
Is this deterioration likely to be septic in nature?
How do you approach the diagnosis of this new episode of likely sepsis?
Nosocomial infection
PACT module on Severe infection
Is adrenal dysfunction a possibility? What do you do?
Adrenal dysfunction in sepsis
PACT module on Homeostasis
PACT module on Hypotension
Hydrocortisone in septic shock
With increasing vasopressor therapy you have obtained a MAP of 70 mmHg which you
consider acceptable and you decide to insert a pulmonary artery catheter which was
uneventful. Measurements reveal a hyperdynamic circulation (CI 4.8 l/min/m 2 and SVRI 900
dynes/sec/cm–5/m2) and a wedge pressure (PAOP) of 16 mmHg. This virtually rules out
hypovolaemia and cardiac output seems adequate. You give 100 mg hydrocortisone i.v., to
be repeated after 8 and 16 hours. However, urine output is still low (20 ml/hr) and serum
creatinine has risen to 180 µmol/l (2.0 mg/dl).
Learning issues
Prevention of acute renal failure in sepsis
What can you do to prevent further deterioration of renal function and acute kidney injury
(AKI)?
N OTE
Prevention
of acute
renal
failure is of
utmost
importance
since
mortality
in sepsis
markedly
increases
when ARF
complicates
its course.
N OTE
In sepsis renal blood flow becomes highly dependent on mean arterial pressure. The
precise mean arterial pressure target is difficult to predict but is typically >80 mmHg
What is the nature of the renal dysfunction and how would you manage the
norepinephrine therapy?
N OTE
Keep in mind
that
mechanisms
other than
hypoperfusion
may affect
renal function
in sepsis. This
should not
discourage
you from
optimising
renal
perfusion
pressure.
You know from textbooks that prerenal dysfunction can be assessed by measuring urinary
sodium concentration and osmolality. Lab results show a urine osmolality of 700 mosmol/l
and urine sodium concentration of 15 mmol/l.
N OTE
These tests have a poor specificity and sensitivity and have not been validated in critically
ill patients.
Learning issues
PACT module on Oliguria and anuria
What is your interpretation of these results?
N OTE
The kidney remains vulnerable to episodes of hypotension also in the
recovery phase.
Do you reduce the dose of penicillin now that the plasma creatinine level has
increased to 180 µmol/l (2.0 mg/dl)? Explain your answer.
N OTE
Always consider potential adverse effects of treatment on renal (and other organ)
function.
You increase the dose of norepinephrine which results in a MAP of 85 mmHg without a
major change in cardiac output. Diuresis starts to increase slightly (35 ml/hr). You feel that
you have now optimised haemodynamics in your patient. In the meantime you are
addressing another real clinical problem: the rise in body temperature. The approach that
may be used can be helpful in all situations where new signs or symptoms appear.
How do you approach the diagnosis of the new fever?
Learning issues
Causes and evaluation of fever in critically ill patients
PACT module on Pyrexia
What are the possible 'missed diagnoses' in this patient?
N OTE
In this septic MODS patient recurrence of fever is a sign of (an
exacerbation of) an inflammatory process unless another cause is
established. This could significantly contribute to further
deterioration of organ function and therefore needs urgent
diagnosis and treatment. Radiologic studies have a high priority
here since they are rapidly obtained and may reveal (or rule out) a
diagnosis.
CXR shows resolution of the pulmonary infiltrates but there is an
increased opacity at the right side compatible with pleural fluid as shown
in the figure on the left. In retrospect, daily chest X-rays show the
gradual development of pleural fluid partly obscured by dense infiltrates.
A sinus X-ray series shows no opacities.
What is your next step following this finding?
The diagnostic pleural puncture reveals yellow-stained turbid fluid. A Gram stain shows
many polymorphonuclear leukocytes and sparsely some Gram-positive cocci.
Chemistry: protein 20 g/l, pH 7.10 (blood 7.42), LDH 1150/U/l (blood 210/U/l), glucose 3.8
mmol/l (blood 6.4 mmol/l).
What is your conclusion and what should be done?
PACT module on Communication skills
You make preparations to insert a drainage tube. The patient has thrombocytopenia (45 x
109/l) but only mildly increased clotting times. The nurse asks you whether platelet
concentrate should be ordered.
How do you respond to the nurse?
Learning issues
Invasive procedures in thrombocytopenia
PACT module on Bleeding and thrombosis
A chest tube is inserted and 1.5 l turbid fluid drained from the right pleural space which is
flushed twice daily with sterile normal saline. After chest tube drainage and treatment with
hydrocortisone, on day six the patient becomes haemodynamically stable allowing for
weaning from vasopressor support. Prophylactic subcutaneous low molecular weight
heparin (LMWH) is continued as prophylaxis against thromboembolism.
What could be the cause of the increasing thrombocytopenia on day four in this
patient?
Learning issues
Thrombocytopenia in sepsis
PACT module on Bleeding and thrombosis
N OTE
Failure of platelet counts to rise during treatment of sepsis should raise suspicions of
missed septic foci ('missed diagnosis') or inadequate (antibiotic) treatment.
Learning issues
Disseminated coagulopathy in sepsis
Would you consider a platelet transfusion in this patient? Explain your answer.
PACT module on Bleeding and thrombosis
N OTE
The best treatment of sepsis-induced thrombocytopenia is adequate treatment of
the septic process.
What pharmacological measures can you consider in order to
prevent further renal injury in your patient?
Learning issues
PACT module on Oliguria and anuria
PACT module on Acute renal failure
N OTE
When potentially nephrotoxic drugs cannot be avoided, plasma levels are
monitored to ensure efficacy of the agent and to minimise toxicity.
In the following days your patient further improves. Body temperature normalises, platelet
count trends upward and a short polyuric phase (without frusemide) precedes a
normalisation of renal function. Cultures of blood and pleural fluid taken on day four show
no growth.
In the meantime, the patient is successfully weaned from mechanical ventilation and
haemodynamic support including hydrocortisone. The chest tube is removed after seven
days and a CXR shows only a residual thickening of the right pleura. The patient is
discharged from the ICU 12 days after admission in a relatively good condition.
Learning issues
Possible role of frusemide (furosemide)
PACT module on Oliguria and anuria
A 53-year-old man with a history of non-insulin-dependent diabetes (type II)
underwent a right hemicolectomy for cancer four days ago. You are called to the
surgical floor to evaluate him because of confusion and new onset of generalised abdominal
pain. His blood pressure is 70/40 mmHg, his heart rate 136 bpm, respiratory rate 28/min,
and temperature 38.6 °C. A Foley (urinary) catheter is inserted, and drains a small amount
of concentrated urine; a transcutaneous oxygen saturation monitor records an SaO 2 of
98%.
Learning issues
Recognising sepsis in the critically ill patient
Cardiovascular support
What are your immediate priorities?
PACT module on Haemodynamic monitoring
As the fluid is being administered, you insert a subclavian line, and record a central venous
pressure of 2 mmHg; the ScvO2 is 52%. You check the blood glucose patient record and it
is normal.
What is the most likely diagnosis and the differential diagnosis?
Learning issues
Diagnosing infection in the acutely ill patient
PACT module on Abdominal problems
Because of this, you continue resuscitative measures and you draw blood for culture, and
start systemic antibiotic therapy with a carbapenem.
Learning issues
Managing infection in the septic patient
Instituting antibiotic therapy
PACT module on Severe infection
What is his pressure-adjusted rate (PAR) at this time?
This value is within normal limits, and, in conjunction with a low MAP, suggest a functional
intravascular volume deficit that may respond to fluids.
N OTE
Pending knowledge of the offending organism(s), you should know your local hospital
formulary recommendations for new onset abdominal sepsis and for the appropriate
escalation of therapy when earlier antibiotic therapy has already been in place.
Learning issues
PAR – a derived index for circulatory assessment
By the time the patient reaches ICU, 3.5 litres of crystalloid have been infused and the heart
rate has dropped to 100, his blood pressure has increased to 110/80 mmHg and his CVP is
now 9 mmHg.
What would his PAR now be and how do you interpret the result?
Despite continued resuscitation and a total of 5 litres of crystalloids, he becomes
tachycardic again at 120/min and hypotensive with a blood pressure of 80/50, and the urine
output has been 30 ml urine over the past hour. The central venous pressure has now
increased to 12, and the SaO2 on supplemental oxygen by mask is 93%. The ScvO2 is 58%.
What are your priorities in the ICU?
Because of a falling SaO2, a reduced level of consciousness, and a need for ongoing
resuscitation, you elect to intubate him, and initiate mechanical ventilation. Blood work is
sent, and an abdominal CT arranged.
He is now on an FiO2 of 0.6, with 12.5 cmH2O of PEEP and the arterial blood gas shows a
PaO2 of 80 mmHg (10.6 kPa). Blood work shows:
 Haemoglobin of 8.7 mmol/l (14.0 g/dl)
 White count of 18.4
 Platelet count of 140 000
 Lactate of 4.2 mmol/l (37.8 mg/dl)
 Blood sugar of 18.6 mmol/l (335 mg/dl)
 Creatinine of 229 µmol/l (2.5 mg/dl)
 Bilirubin of 21 µmol/l (1.2 mg/dl).
Stabilisation appears not to be possible until the cause of his acute deterioration has been
addressed.
Learning issues
PACT module on Mechanical ventilation
What are the priorities in terms of 'source control'?
What is his MOD score at this time?
He is transferred to the operating room, and at laparotomy is found to have a dehiscence of
his anastomosis, with diffuse purulent peritonitis. The abdomen is irrigated, and the
disrupted anastomosis exteriorised as an ileostomy and a mucous fistula; the abdomen is
closed primarily. He receives a further 6.5 litres of fluid intraoperatively, and is maintained
on an epinephrine infusion.
Over the next six hours following his transfer back to the ICU, his urine output decreases,
his creatinine rises to 279 µmol/l (3.1 mg/dl), and his tidal volume decreases. The PIP (and
pPlat) continue to increase and the ventilator is constantly alarming. The dose of
dobutamine and norepinephrine has increased, and his haemodynamic parameters reveal a
blood pressure of 80/50, heart rate 140, CVP of 22 mmHg, and ScvO2 of 62. His PAR at
this time is now over 30.
Learning issues
Source control
How do you interpret his deterioration? What should be done?
Learning issues
PACT module on Oliguria and anuria
PACT module on Abdominal problems
The Foley catheter is transduced, and the estimated intra-abdominal pressure is 42 mmHg.
He is taken back to the operating room, and the incision reopened, achieving temporary
closure with an absorbable mesh. His cardiorespiratory status improves.
It is now the morning of the next day. He remains on norepinephrine and dobutamine, but
the ScVO2 is now 74%, and he is on an FiO2 of 0.5, with a PaO2 of 80 mmHg (10.6 kPa).
HR is 102 and BP is now stable at 110/74.
Learning issues
Intra-abdominal pressure
PACT module on Abdominal problems
Are there adjunctive therapies that should be considered at this stage?
Learning issues
Adjunctive therapy for sepsis
Because he continues to remain febrile, vasopressor dependent, and his platelet count is
dropping, but is not low enough to contraindicate protein C, activated protein C (aPC) is
started later on.
Are there other (supportive) measures that you maintain and review at this stage?
PACT module on Nutrition
PACT module on Abdominal problems
http://www.survivingsepsis.org/
He remains intubated and sedated over the next five days. His MOD score has only fallen
marginally to 11, having been 14 at admission. He continues to have a low grade fever on
antibiotics; the organisms from his peritoneal pus (taken at surgery) are covered by the
current antimicrobial therapy and there is no clinical evidence of nosocomial pneumonia and
sputum cultures are sterile.
What else would you do?
The CT scan confirms the presence of a large pelvic abscess. Judicious exploration of the
wound at the bedside by the surgeon and yourself allows entry to the collection, and several
hundred ml of foul-smelling purulent material is drained. Had this not been possible, you
would have opted for percutaneous drainage. Following drainage of this pelvic abscess, the
patient improves. He is weaned from vasoactive support, and a tracheostomy is performed.
The platelet count rises. Urine output increases, and as he begins to mobilise fluid, his level
of consciousness improves. He is weaned from the ventilator during the fourth ICU week,
and discharged to the ward two days later.
Learning issues
Managing infection – drainage
Learning issues
PACT module on Airway management
Learning issues
PACT module on Mechanical ventilation
On reflection, rapid diagnosis and treatment is essential in the clinical management of sepsis. Both cases
illustrate the dual approach involving concurrent resuscitation/organ support and specific therapy targeted to
the source of the infection/sepsis. Careful clinical evaluation is required for diagnostic precision from the
outset and to allow ongoing vigilance to prevent secondary or recurrent problems which are frequent in
patients with sepsis.
Q1. Management of septic shock due to colonic anastomotic breakdown
includes
Your answers
A. Prompt administration of appropriate antibiotics
The correct answer is : True
B. Targeting an ScvO2 of 60% or more
The correct answer is : False
C. Repeat laparotomy if the intra-abdominal pressure measured via a Foley (bladder)
catheter equals 15 mmHg
The correct answer is : False
D. Immediate administration of activated protein C
The correct answer is : False
E. Low-dose hydrocortisone in those with fluid- and vasopressor-resistant hypotension
The correct answer is : True
Your total score is 0/5
Q2. In an adult patient, the following are clinical features of the early
course of severe sepsis
Q3. The following are mechanisms responsible for tissue hypoxia in
severe sepsis
Your answers
A. Relative hypovolaemia causing hypotension
The correct answer is : True
B. Maldistribution of blood flow because of microvascular thrombosis
The correct answer is : True
C. Maldistribution of blood flow because of arterio-venous shunting
The correct answer is : True
D. Loss of intravascular fluid due to capillary leakage
The correct answer is : True
E. Myocardial depression due to coronary vasospasm
The correct answer is : False
Your total score is 0/5
Q4. Optimal timing for initial interventions after recognition of septic shock is
Your answers
A. Within one hour for intravenous corticosteroids
The correct answer is : False - Corticosteroids in this setting are an adjunctive therapy;
they are recommended if vasopressors are needed after adequate fluid resuscitation.
Corticosteroids are not a resuscitative intervention.
B. Within one hour to obtain adequate cultures
The correct answer is : True
C. Within six hours for aggressive fluid resuscitation
The correct answer is : False - Fluid resuscitation of a patient in septic shock should
begin immediately after recognition of this clinical situation
D. Within 12 hours for vasopressors after fluid resuscitation
The correct answer is : False - Vasopressors, to restore adequate blood pressure,
should be started whenever needed, as soon as possible, to complement fluid
resuscitation
E. Within one hour for intravenous antibiotic therapy against the likely pathogen
The correct answer is : True - Effective antimicrobial therapy should be started within
one hour: delay between start of hypotension and antibiotics is a determinant of
survival
Your total score is 0/5
Q5. A 64-year-old man has been treated for necrotising pancreatitis
with septic shock and multiorgan dysfunction including acute renal
failure. It takes three weeks of treatment before stabilisation is
achieved and weaning from mechanical ventilation can be considered.
Two days after stopping all sedation the patient remains drowsy and
unresponsive to command. He is on pressure support ventilation with
adequate oxygenation and ventilation. Tone and peripheral reflexes are
normal. The preferred strategy in this condition is
Your answers
A. Performing a CT-scan of the brain
The correct answer is : False - CT not indicated as there are no lateralising signs and
septic/drug-related encephalopathy is the most likely
B. Ordering an EMG
The correct answer is : False - Not indicated as tone and reflexes are normal and other
explanation (Answer A above) most likely
C. Expectant
The correct answer is : True - Reasonable to allow time for septic encephalopathy to
resolve and elimination of drug effect to occur especially in light of the renal failure
which can impair opiate (especially morphine metabolite) elimination
D. Starting an infusion of naloxone
The correct answer is : False - Not indicated, unless given very cautiously in the
diagnosis of possible excess opiate effect. Normally benefit of such intervention would
not outweigh the possible adverse effect
E. Starting an infusion of flumazenil
The correct answer is : False - Not indicated, unless given very cautiously in the
diagnosis of possible excess benzodiazepine effect. Normally benefit of such
intervention would not outweigh the possible adverse effect
Your total score is 0/5